Anda di halaman 1dari 28

Digested Cases in Labor Law 1

Atty. Carlisle Marie Anselyn Fabie

LVN PICTURES, INC. vs. PHILIPPINE MUSICIANS Guild

FACTS:

Respondent Philippine Musicians Guild (FFW) is a duly registered legitimate labor organization. LVN Pictures, Inc., Sampaguita
Pictures, Inc., and Premiere Productions, Inc. are corporations, duly organized under the Philippine laws, engaged in the making of motion
pictures and in the processing and distribution thereof. Petitioner companies employ musicians for the purpose of making
music recordings for title music, background music, musical numbers, finale music and other incidental music, without which a
motion picture is incomplete. Ninety-five(95%) percent of all the musicians playing for the musical recordings of said companies
are members of the Guild. The Guild has no knowledge of the existence of any other legitimate labor organization representing
musicians in said companies. Premised upon these allegations, the Guild prayed that it be certified as the sole and exclusive
bargaining agency for all musicians working in the aforementioned companies. In their respective answers, the latter denied
that they have any musicians as employees, and alleged that the musical numbers in the filing of the companies are furnished by independent
contractors. The lower court sustained the Guild’s theory. Are consideration of the order complained of having been denied by the Court en
banc, LVN Pictures, Inc., and Sampaguita Pictures, Inc., filed these petitions for review for certiorari.

ISSUE:

Whether the musicians in question (Guild members) are “employees “of the petitioner film companies.

HELD:

YES. The Court agreed with the lower court’s decision, to wit: Lower court resorted to apply R.A. 875 and US Laws and jurisprudence from which
said Act was patterned after. (Since statutes are to be construed in the light of purposes achieved and the evils sought to be
remedied). It ruled that the work of the musical director and musicians is a functional and integral part of the enterprise performed at the same
studio substantially under the direction and control of the company. In other words, to determine whether a person who
performs work for another is the latter's employee or an independent contractor, the National Labor Relations relies on 'the
right to control' test. Under this test an employer-employee relationship exist where the person for whom the services are
performed reserves the right to control not only the end to be achieved, but also the manner and means to be used in reaching
the end. (United Insurance Company, 108, NLRB No. 115.). Notwithstanding that the employees are called independent
contractors', the Board will hold them to be employees under the Act where the extent of the employer's control over them
indicates that the relationship is in reality one of employment. (John Hancock Insurance Co., 2375-D, 1940, Teller, Labor Dispute
Collective Bargaining, Vol.).

The right of control of the film company over the musicians is shown:

(1) by calling the musicians through 'call slips' in 'the name of the company;
(2) by arranging schedules in its studio for recording sessions;
(3) by furnishing transportation to musicians; and
(4) by supervising and directing in detail, through the motion picture director, the performance of the musicians before the
camera, in order to suit the music they are playing to the picture which is being flashed on the screen.

The “musical directors” have no such control over the musicians involved in the present case. Said musical directors control
neither the music to be played, nor the musicians playing it. The Premier Production did not appeal the decision of the Court en
banc (that’s why it’s not one of the petitioners in the case) film companies summon the musicians to work, through the musical
directors. The film companies, through the musical directors, fix the date, the time and the place of work. The film companies,
not the musical directors, provide the transportation to and from the studio. The film companies furnish meal at dinner time. It
is well settled that "an employer-employee relationship exists . . .where the person for whom the services are performed reserves
a right to control not only the end to be achieved but also the means to be used in reaching such end . . . ." The decisive nature of said
control over the "means to be used", is illustrated in the case of Gilchrist Timber Co., et al., in which, by reason of said control,
the employer-employee relationship was held to exist between the management and the workers, notwithstanding the
intervention of an alleged independent contractor, who had, and exercise, the power to hire and fire said workers. The
aforementioned control over the means to be used" in reading the desired endis possessed and exercised by the film companies
over the musicians in the cases before us. WHEREFORE, the order appealed from is hereby affirmed, with costs against petitioners
herein. It is so ordered.

DY KEH BENG vs. INTERNATIONAL LABOR and MARITIME UNION

FACTS:

Petitioner, Dy Keh Beng, proprietor of basket factory, was charged with ULP for discriminatory acts defined under Sec 4(a),
subparagraph (1 & 4), R.A. No. 875 by dismissing on September 28-29, 1960, respectively, Carlos N. Solano and Ricardo Tudla for
their union activities. After PI was conducted, a case was filed in the CIR for in behalf of the ILMUP and two of its members, Solano
and Tudla. Dy Keh Beng contended that he did not know Tudla and that Solano was not his employee because the latter came
tothe establishment only when there was work which he did on pakiaw basis. According to Dy Keh Beng, Solano was nothis
employee for the following reasons:(1) Solano never stayed long enough at Dy’s establishment;(2) Solano had to leave as soon
as he was through with the order given him by Dy;(3) When there were no orders needing his services there was nothing for him
to do;(4) When orders came to the shop that his regular workers could not fill it was then that Dy went to his address in Caloocan
and fetched him for these orders; and(5) Solano's work with Dy's establishment was not continuous.

According to petitioner, these facts show that respondents Solano and Tudla are only piece workers, not employees
under Republic Act 875, where an employee is referred to as shall include any employee and shag not be limited to the employee
of a particular employer unless the act explicitly states otherwise and shall include any individual whose work has ceased as a
consequence of, or in connection with any current labor dispute or because of any ulp and who has not obtained any other
substantially equivalent and regular employment while an employer includes any person acting in the interest of an employer,
directly or indirectly but shall not include any labor organization (otherwise than when acting as an employer) or anyone acting
in the capacity of officer or agent of such labor organization Petitioner also contends that the private respondents "did not meet
the control test in the fight of the ... definition of the terms employer and employee, because there was no evidence to show
that petitioner had the right to direct the manner and method of respondent's work.

He points to the case of Shipping Co., Inc. v. Nieves Baens del Rosario, et al.,L-13130, October 31, 1959, where the Court ruled that: The test
... of the existence of employee and employer relationship is whether there is an understanding parties that one is to render
personal services to or for the benefit of the other and recognition by them of the right of one to order and control the other in
the performance of the work and to direct the manner and method of its performance. The CIR found that there existed an
employee-employer relationship between Dy Keh Beng and complainants Tudla and Solano, although Solano was admitted to
have worked on piece basis. Hence, this petition for certiorari.

ISSUE:

Whether or not an employee employer relation existed between petitioner Dy Keh Beng and the respondents Solano and Tudla.

HELD:

Yes. Evidence showed that the work of Solano and Tudla was continuous except in the event of
illness,although their services were compensated on piece basis. The SC noted the decision of Justice Paras in the case of
³Sunrise Coconut Products Co. Vs. CIR (83 Phil 518, 523) that judicial notice of the fact that the so-called "pakyaw" system
mentioned in this case as generally practiced in our country, is, in fact, a labor contract -between employers and employees,
between capitalists and laborers. With regard to the control test the SC said that ³It should be borne in mind that the control
test calls merely for the existence of the right to control the manner of doing the work, not the actual exercise of the right.
Considering the finding by the Hearing Examiner that the establishment of Dy Keh Beng is "engaged in the manufacture of baskets
known as kaing it is natural to expect that those working under Dy would have to observe, among others, Dy's requirements of
size and quality of the kaing . Some control would necessarily be exercised by Dy as the making of the kaing would be subject to
Dy's specifications. Parenthetically, since the work on the baskets is done at Dy's establishments, it can be inferred that the
proprietor Dy could easily exercise control on the men he employed. The petition was dismissed. The Court affirmed the decision
of the CIR.

As to the contention that Solano was not an employee because he worked on piece basis, the court ruled that it should be
determined that if indeed payment by piece is just a method of compensation and does not define the essence of the relation.
Payment cannot be construed by piece where work is done in such establishment so as to put the worker completely at liberty
to turn him out and take it another at pleasure Justice Perfecto also contended that pakyaw system is a labor contract between
employers and employees between capitalists and laborers.

CORPORAL SR. VS. NLRC

FACTS:

Five male barbers and 2 female manicurists (Petitioners) worked at New Look Barbershop, a sole proprietorship owned and
managed by Vicente Lao which in 1982 was taken over by Lao Enteng Co., Inc., (respondent corporation) a corporation formed
by Vicente Lao’s children. The petitioners were allowed to work there until April 1985 when they were told that the barbershop
building was sold and their services are no longer needed. Petitioners filed with the Arbitration branch of NLRC a complaint for
illegal dismissal, illegal deduction, separation pay, non-payment of 13th month pay and salary differential. Also they seek for
refund of P1.00 collected from each of them daily as salary of the barbershop’s sweeper. Respondent Corporation alleged that
petitioners were Joint Venture (JV) partners receiving 50% commission (Petitioners admitted in receiving 50-60%), therefore no
employer-employee relationship existed. And assuming arguendothat employer-employee relationship existed, petitioners were
not entitled to separation pay since cessation of the business was due to serious business losses. Also, they allege that the
barbershop had always been a JV partnership with the operation and management left entirely to petitioners and that the former
had no control over the latter who could freely come and go as they wish. Lastly, they allege that some of the petitioners were
allowed to register in SSS only as an act of accommodation. The Labor Arbiter dismissed the complaint and found that there was
a JV and no employer-employee relationship. Also that the business was closed due to serious business losses or financial reverses
and the law does not compel the establishment to pay separation pay to whoever were its employees. On appeal, NLRC affirmed
the decision but held that petitioners were considered independent contractors and not employees. The MR was also denied by
NLRC, hence, this petition on certiorari.

ISSUE:

WON there was an employer-employee relationship.

HELD:

YES.
Petitioners are employees of Respondent Corporation and shall be accorded the benefits given in Art. 283 of the Labor Code
granting separation pay equivalent to 1 month pay for every year of service and also to 13th month pay. The other claims of
petitioners are found to be without basis.

No documentary evidence of the existence of JV other than the self-serving affidavit of the company president.

The power of control in the 4-fold test (employer-employee relationship) refers to the EXISTENCE and NOT THE EXERCISE of
such power the following elements must be present for an employer-employee relationship to exist:

(1) the selection and engagement of the workers;


(2) power of dismissal;
(3) the payment of wages by whatever means; and
(4) the power to control the worker's conduct, with the latter assuming primacy in the overall consideration.

The records show that Vicente Lao engaged the petitioners to work for the barbershop and retained them after it was taken over
by the respondent corporation who continuously paid their wages. Also, the fact that the petitioners worked in the barbershop
owned and operated by respondents, and that they were required to report daily, observing definite hours of work.

PEOPLE’S BROADCASTING (BOMBO RADYO PHILS., INC.) vs THE SECRETARY OF THE DEPARTMENT OF LABOR AND
EMPLOYMENT

FACTS:

Jandeleon Juezan (“Juezan”) filed a complaint before the DOLE against Bombo Radyo Phils. (“Bombo Radyo”) for illegal deduction,
non-payment of service incentive leave, 13th month pay, premium pay for holiday and rest day and illegal diminution of benefits,
delayed payment of wages and non-coverage of SSS, PAG-IBIG and Philhealth. On the basis of the complaint, the DOLE conducted
a plant level inspection. The Labor Inspector in his report wrote,
Management representative informed that (Juezan) complainant is a drama talent hired on a per drama ‘participation
basis’ hence no employer-employer relationship existed between them. As proof of this, management presented
photocopies of cash vouchers, billing statement, employments of specific undertaking, etc. The management has no
control of the talent if he ventures into another contract with other broadcasting industries.

The DOLE Regional Director issued an order ruling that Juezan is an employee of Bombo Radyo, and that Juezan is entitled to his
money claims. Bombo Radyo sought reconsideration claiming that the Regional Director gave credence to the documents offered
by Juezan without examining the originals, but at the same time the Regional Director missed or failed to consider Bombo Radyo’s
evidence. The motion for reconsideration was denied. On appeal, the Acting DOLE Secretary dismissed the appeal on the ground
that Bombo Radyo did not post a cash or surety bond and instead submitted a Deed of Assignment of Bank Deposit.

Bombo Radyo elevated the case to the Court of Appeals, claiming that it was denied due process when the DOLE Secretary
disregarded the evidence it presented and failed to give it the opportunity to refute the claims of Juezan. It maintained that no
employer-employee relationship had ever existed between it and Juezan because it was the drama directors and producers who
paid, supervised and disciplined him. It also added that the case was beyond the DOLE’s jurisdiction because Juezan’s claim
exceeded P5,000.
The Court of Appeals held that the DOLE Secretary had the power to order and enforce compliance with labor standard laws
irrespective of the amount of individual claims because the limitation imposed by Art. 29 of the Labor Code had been repealed
by R.A. 7730.

Bombo Radyo argues that the NLRC (not the DOLE Secretary) has jurisdiction over Juezan’s claim, in view of Arts. 217 and 128 of
the Labor Code. It adds that the Court of Appeals committed grave abuse of discretion when it dismissed their appeal without
delving on the issue of employer-employee relationship.

ISSUE:

Whether or not the Secretary of Labor has the power to determine the existence of an employer-employee relationship.

HELD:

NO. Art. 128 (b) of the Labor Code, as amended by R.A. 7730 reads:

Notwithstanding the provisions of Articles 129 and 217 of this Code to the contrary, and in cases where the
relationship of employer-employee still exists, the Secretary of Labor and Employment or his duly authorized
representatives shall have the power to issue compliance orders to give effect to the labor standards provisions of
this Code and other labor legislation based on the findings of labor employment and enforcement officers or industrial
safety engineers made in the course of inspection.

The provision is explicit that the visitorial and enforcement power of the DOLE comes into play only “in cases when the
relationship of employer-employee still exists.” This clause signifies that the employer-employee relationship must have
existed even before the emergence of the controversy. Necessarily, the DOLE’s power does not apply in two instances, namely:
(i) where the employer-employee relationship has ceased; and (ii) where no such relationship has ever existed.

The first situation is categorically covered by Sec. 3, Rule 11 of the Rules on the Disposition of Labor Standards Cases issued by
the DOLE Secretary. It reads:

Where employer-employee relationship no longer exists by reason of the fact that it has already been severed, claims
for payment of monetary benefits fall within the exclusive and original jurisdiction of the labor arbiters. Accordingly,
if on the face of the complaint, it can be ascertained that employer-employee relationship no longer exists, the case,
whether accompanied by an allegation of illegal dismissal, shall immediately be endorsed by the Regional Director to
the appropriate branch of the National Labor Relations Commission (NLRC).

The law accords a prerogative to the NLRC over the claim when the employer-employee relationship has terminated or such
relationship has not arisen at all. The existence of an employer-employee relationship is a matter which is not easily determinable
from an ordinary inspection because the elements of such a relationship are not verifiable from a mere ocular examination. The
intricacies and implications of an employer-employee relationship demand that the level of scrutiny should be far above the
superficial. While documents, particularly documents found in the employer’s office are the primary source materials, what may
prove decisive are factors related to the history of the employer’s business operations, its current state as well as accepted
contemporary practices in the industry. More often than not, the question of employer-employee relationship becomes a battle
of evidence, the determination of which should be comprehensive and intensive and therefore best left to the specialized quasi-
judicial body of the NLRC.

It can be assumed that the DOLE in the exercise of its visitorial and enforcement power somehow has to make a determination
of the existence of an employer-employee relationship. However, such determination cannot be coextensive with the visitorial
and enforcement power itself. Such is merely preliminary, incidental and collateral to the DOLE’s primary function of enforcing
labor standards provisions. The determination of the existence of employer-employee relationship is still primarily lodged with
the NLRC. This is the meaning of the clause “in cases where the relationship of employer-employee still exists” in Art. 128 (b).

Thus, before the DOLE may exercise its powers under Art. 128, two important questions must be resolved: (i) Does the employer-
employee relationship still exist, or alternatively, was there ever an employer-employee relationship to speak of; and (ii) Are
there violations of the Labor Code or of any labor law?

The existence of an employer-employee relationship is a statutory prerequisite to and a limitation on the power of the Secretary
of Labor, one which the legislative branch is entitled to impose. The rationale underlying this limitation is to eliminate the prospect
of competing conclusions of the Secretary of Labor and the NLRC. If the Secretary of Labor proceeds to exercise his visitorial and
enforcement powers absent the first requisite, his office confers jurisdiction on itself which it cannot otherwise acquire.

Nevertheless, a mere assertion of absence of employer-employee relationship does not deprive the DOLE of jurisdiction over the
claim. At least a prima facie showing of such absence of relationship, as in this case, is needed to preclude the DOLE from the
exercise of its power. Without a doubt, Bombo Radyo, since the inception of this case had been consistent in maintaining that
Juezan is not its employee. A preliminary determination, based on the evidence offered and noted by the Labor Inspector during
the inspection as well as submitted during the proceedings before the Regional Director puts in genuine doubt the existence of
employer-employee relationship. From that point on, the prudent recourse on the part of the DOLE should have been to refer
Juezan to the NLRC for the proper dispensation of his claims. Furthermore, even the evidence relied on by the Regional Director
in his order are mere self-serving declarations of Juezan, and hence cannot be relied upon as proof of employer-employee
relationship.

FRANCISCO VS NLRC

FACTS:

1995, Petitioner was hired by Kasei Corporation during its incorporation stage. She was designated as Accountant and Corporate
Secretary and was assigned to handle all the accounting needs of the company. She was also designated as Liaison Officer to the
City of Makati to secure business permits, construction permits and other licenses for the initial operation of the company.

Although she was designated as Corporate Secretary, she was not entrusted with the corporate documents; neither did she
attend any board meeting nor required to do so. She never prepared any legal document and never represented the company
as its Corporate Secretary. 1996, petitioner was designated Acting Manager. Petitioner was assigned to handle recruitment of all
employees and perform management administration functions; represent the company in all dealings with government agencies,
especially with the BIR, SSS and in the city government of Makati; and to administer all other matters pertaining to the operation
of Kasei Restaurant which is owned and operated by Kasei Corporation.

January 2001, petitioner was replaced by a certain Liza R. Fuentes as Manager. Kasei Corporation reduced her salary, she was not
paid her mid-year bonus allegedly because the company was not earning well. On October 2001, petitioner did not receive her
salary from the company. She made repeated follow-ups with the company cashier but she was advised that the company was
not earning well. Eventually she was informed that she is no longer connected with the company.

Since she was no longer paid her salary, petitioner did not report for work and filed an action for constructive dismissal before
the labor arbiter. Private respondents averred that petitioner is not an employee of Kasei Corporation. They alleged that
petitioner was hired in 1995 as one of its technical consultants on accounting matters and act concurrently as Corporate
Secretary. As technical consultant, petitioner performed her work at her own discretion without control and supervision of Kasei
Corporation. Petitioner had no daily time record and she came to the office any time she wanted and that her services were only
temporary in nature and dependent on the needs of the corporation.

The Labor Arbiter found that petitioner was illegally dismissed, NLRC affirmed with modification the Decision of the Labor Arbiter.
On appeal, CA reversed the NLRC decision. CA denied petitioner’s MR, hence, the present recourse.

ISSUES:

1. WON there was an employer-employee relationship between petitioner and private respondent; and if in the
affirmative,
2. Whether petitioner was illegally dismissed.

RULING:

1. Generally, courts have relied on the so-called right of control test where the person for whom the services are
performed reserves a right to control not only the end to be achieved but also the means to be used in reaching such
end. In addition to the standard of right-of-control, the existing economic conditions prevailing between the parties,
like the inclusion of the employee in the payrolls, can help in determining the existence of an employer-employee
relationship.

There are instances when, aside from the employer’s power to control the employee, economic realities of the employment
relations help provide a comprehensive analysis of the true classification of the individual, whether as employee, independent
contractor, corporate officer or some other capacity.

It is better, therefore, to adopt a two-tiered test involving: (1) the employer’s power to control; and (2) the economic realities
of the activity or relationship.

The control test means that there is an employer-employee relationship when the person for whom the services are performed
reserves the right to control not only the end achieved but also the manner and means used to achieve that end.

There has to be analysis of the totality of economic circumstances of the worker. Thus, the determination of the relationship
between employer and employee depends upon the circumstances of the whole economic activity, such as:

(1) the extent to which the services performed are an integral part of the employer’s business;
(2) the extent of the worker’s investment in equipment and facilities;
(3) the nature and degree of control exercised by the employer;
(4) the worker’s opportunity for profit and loss;
(5) the amount of initiative, skill, judgment or foresight required for the success of the claimed independent enterprise;
(6) the permanency and duration of the relationship between the worker and the employer; and
(7) the degree of dependency of the worker upon the employer for his continued employment in that line of business.

The proper standard of economic dependence is whether the worker is dependent on the alleged employer for his continued
employment in that line of business.

By applying the control test, it can be said that petitioner is an employee of Kasei Corporation because she was under the direct
control and supervision of Seiji Kamura, the corporation’s Technical Consultant. She reported for work regularly and served in
various capacities as Accountant, Liaison Officer, Technical Consultant, Acting Manager and Corporate Secretary, with
substantially the same job functions, that is, rendering accounting and tax services to the company and performing functions
necessary and desirable for the proper operation of the corporation such as securing business permits and other licenses over an
indefinite period of engagement. Respondent corporation had the power to control petitioner with the means and methods by
which the work is to be accomplished.

Under the economic reality test, the petitioner can also be said to be an employee of respondent corporation because she had
served the company for 6 yrs. before her dismissal, receiving check vouchers indicating her salaries/wages, benefits, 13th month
pay, bonuses and allowances, as well as deductions and Social Security contributions from. When petitioner was designated
General Manager, respondent corporation made a report to the SSS. Petitioner’s membership in the SSS evinces the existence
of an employer-employee relationship between petitioner and respondent corporation. The coverage of Social Security Law is
predicated on the existence of an employer-employee relationship.

2. The corporation constructively dismissed petitioner when it reduced her. This amounts to an illegal termination of
employment, where the petitioner is entitled to full backwages

A diminution of pay is prejudicial to the employee and amounts to constructive dismissal. Constructive dismissal is an involuntary
resignation resulting in cessation of work resorted to when continued employment becomes impossible, unreasonable or
unlikely; when there is a demotion in rank or a diminution in pay; or when a clear discrimination, insensibility or disdain by an
employer becomes unbearable to an employee. Petition is GRANTED.

JAVIER vs FLY ACE CORPORATION

FACTS:

Javier an employee of Fly Ace performing various work for the latter filed a complaint before the NLRC for underpayment of
salaries and other labor standard benefits. He alleged that he reported for work from Monday to Saturday from 7:00 oclock in
the morning to 5:00 o clock in the afternoon; that during his employment, he was not issued an identification card and pay slips
by the company; that he reported for work but he was no longer allowed to enter the company premises by the security guard
upon the instruction of Ruben Ong (Mr. Ong), his superior; that after several minutes of begging to the guard to allow him to
enter, he saw Ong whom he approached and asked why he was being barred from entering the premises; that Ong replied by
saying, Tanungin mo anak mo; that he discovered that Ong had been courting his daughter Annalyn after the two met at a fiesta
celebration in Malabon City; that Annalyn tried to talk to Ong and convince him to spare her father from trouble but he refused
to accede; that thereafter, Javier was terminated from his employment without notice; and that he was neither given the
opportunity to refute the cause/s of his dismissal from work. For its part p, Fly Ace denied the existence of employer-employee
relationship between them and Javier as the latter was only called roughly 5 to 6 times only in a month whenever the vehicle of
its contracted hauler, Milmar Hauling Services, was not available. Labor Arbiter dismissed the complaint ruling that respondent
Fly Ace is not engaged in trucking business but in the importation and sales of groceries. Since there is a regular hauler to deliver
its products, we give credence to Respondents claim that complainant was contracted on pakiao basis. On appeal, NLRC reversed
the decisin of the LA. It was of the view that a pakyaw- basis arrangement did not preclude the existence of employer-employee
relationship. Payment by result x x x is a method of compensation and does not define the essence of the relation. It is a mere
method of computing compensation, not a basis for determining the existence or absence of an employer-employee relationship.
The NLRC further averred that it did not follow that a worker was a job contractor and not an employee, just because the work
he was doing was not directly related to the employers trade or business or the work may be considered as extra helper as in this
case; and that the relationship of an employer and an employee was determined by law and the same would prevail whatever
the parties may call it. Finding Javier to be a regular employee, the NLRC ruled that he was entitled to a security of tenure. For
failing to present proof of a valid cause for his termination, Fly Ace was found to be liable for illegal dismissal of Javier who was
likewise entitled to backwages and separation pay in lieu of reinstatement. However, on appeal, CA reversed the ruling of NLRC
The CA ruled thatJaviers failure to present salary vouchers, payslips, or other pieces of evidence to bolster his contention, pointed
to the inescapable conclusion that he was not an employee of Fly Ace. Further, it found that Javiers work was not necessary and
desirable to the business or trade of the company, as it was only when there were scheduled deliveries, which a regular hauling
service could not deliver, that Fly Ace would contract the services of Javier as an extra helper. Lastly, the CA declared that the
facts alleged by Javier did not pass the control test. He contracted work outside the company premises; he was not required to
observe definite hours of work; he was not required to report daily; and he was free to accept other work elsewhere as there
was no exclusivity of his contracted service to the company, the same being co-terminous with the trip only. Since no substantial
evidence was presented to establish an employer-employee relationship, the case for illegal dismissal could not prosper. Hence,
this appeal.

ISSUE:

Does an employer-employee relationship exist between Javier and Fly Ace, thereby holding the latter guilty of illegal dismissal?

HELD:

The Court is of the considerable view that on Javier lies the burden to pass the well-settled tests to determine the existence of
an employer-employee relationship, viz:
(1) the selection and engagement of the employee;
(2) the payment of wages;
(3) the power of dismissal; and
(4) the power to control the employee’s conduct.

Of these elements, the most important criterion is whether the employer controls or has reserved the right to control the
employee not only as to the result of the work but also as to the means and methods by which the result is to be accomplished.

As the records bear out, the LA and the CA found Javier’s claim of employment with Fly Ace as wanting and deficient. The Court
is constrained to agree. Although Section 10, Rule VII of the New Rules of Procedure of the NLRC allows a relaxation of the rules
of procedure and evidence in labor cases, this rule of liberality does not mean a complete dispensation of proof. Labor officials
are enjoined to use reasonable means to ascertain the facts speedily and objectively with little regard to technicalities or
formalities but nowhere in the rules are they provided a license to completely discount evidence, or the lack of it. The quantum
of proof required, however, must still be satisfied. Hence, “when confronted with conflicting versions on factual matters, it is for
them in the exercise of discretion to determine which party deserves credence on the basis of evidence received, subject only to
the requirement that their decision must be supported by substantial evidence.” Accordingly, the petitioner needs to show by
substantial evidence that he was indeed an employee of the company against which he claims illegal dismissal.

Expectedly, opposing parties would stand poles apart and proffer allegations as different as chalk and cheese. It is, therefore,
incumbent upon the Court to determine whether the party on whom the burden to prove lies was able to hurdle the same. “No
particular form of evidence is required to prove the existence of such employer-employee relationship. Any competent and
relevant evidence to prove the relationship may be admitted. Hence, while no particular form of evidence is required, a finding
that such relationship exists must still rest on some substantial evidence. Moreover, the substantiality of the evidence depends
on its quantitative as well as its qualitative aspects.” Although substantial evidence is not a function of quantity but rather of
quality, the x x x circumstances of the instant case demand that something more should have been proffered. Had there been
other proofs of employment, such as x x x inclusion in petitioner’s payroll, or a clear exercise of control, the Court would have
affirmed the finding of employer-employee relationship.”

In sum, the rule of thumb remains: the onus probandi falls on petitioner to establish or substantiate such claim by the requisite
quantum of evidence. “Whoever claims entitlement to the benefits provided by law should establish his or her right
thereto” Sadly, Javier failed to adduce substantial evidence as basis for the grant of relief.

In this case, the LA and the CA both concluded that Javier failed to establish his employment with Fly Ace. By way of evidence on
this point, all that Javier presented were his self-serving statements purportedly showing his activities as an employee of Fly
Ace. Clearly, Javier failed to pass the substantiality requirement to support his claim. Hence, the Court sees no reason to depart
from the findings of the CA.

SONZA vs ABS CBN CORPORATION

FACTS:

In May 1994, ABS-CBN signed an agreement with the Mel and Jay Management and Development Corporation (MJMDC). ABS-
CBN was represented by its corporate officers while MJMDC was represented by Sonza, as President and general manager, and
Tiangco as its EVP and treasurer. Referred to in the agreement as agent, MJMDC agreed to provide Sonza’s services exclusively
to ABS-CBN as talent for radio and television. ABS-CBN agreed to pay Sonza a monthly talent fee of P310, 000 for the first year
and P317, 000 for the second and third year.

On April 1996, Sonza wrote a letter to ABS-CBN where he irrevocably resigned in view of the recent events concerning his program
and career. After the said letter, Sonza filed with the Department of Labor and Employment a complaint alleging that ABS-CBN
did not pay his salaries, separation pay, service incentive pay,13th month pay, signing bonus, travel allowance and amounts under
the Employees Stock Option Plan (ESOP). ABS-CBN contended that no employee-employer relationship existed between the
parties. However, ABS-CBN continued to remit Sonza’s monthly talent fees but opened another account for the same purpose.

The Labor Arbiter dismissed the complaint and found that there is no employee-employer relationship. NLRC affirmed the
decision of the Labor Arbiter. CA also affirmed the decision of NLRC.
ISSUE:

Whether or not there was employer-employee relationship between the parties.

HELD:

Case law has consistently held that the elements of an employee-employer relationship are selection and engagement of the
employee, the payment of wages, the power of dismissal and the employer’s power to control the employee on the means and
methods by which the work is accomplished. The last element, the so-called "control test", is the most important element.

Sonza’s services to co-host its television and radio programs are because of his peculiar talents, skills and celebrity status.
Independent contractors often present themselves to possess unique skills, expertise or talent to distinguish them from ordinary
employees. The specific selection and hiring of SONZA, because of his unique skills, talent and celebrity status not possessed by
ordinary employees, is a circumstance indicative, but not conclusive, of an independent contractual relationship. All the talent
fees and benefits paid to SONZA were the result of negotiations that led to the Agreement. For violation of any provision of the
Agreement, either party may terminate their relationship. Applying the control test to the present case, we find that SONZA is
not an employee but an independent contractor.

The control test is the most important test our courts apply in distinguishing an employee from an independent contractor.
This test is based on the extent of control the hirer exercises over a worker. The greater the supervision and control the hirer
exercises, the more likely the worker is deemed an employee. The converse holds true as well – the less control the hirer
exercises, the more likely the worker is considered an independent contractor. To perform his work, SONZA only needed his
skills and talent. How SONZA delivered his lines, appeared on television, and sounded on radio were outside ABS-CBN’s control.
ABS-CBN did not instruct SONZA how to perform his job. ABS-CBN merely reserved the right to modify the program format and
airtime schedule "for more effective programming." ABS-CBN’s sole concern was the quality of the shows and their standing in
the ratings.

Clearly, ABS-CBN did not exercise control over the means and methods of performance of Sonza’s work. A radio broadcast
specialist who works under minimal supervision is an independent contractor. Sonza’s work as television and radio program host
required special skills and talent, which SONZA admittedly possesses.

ABS-CBN claims that there exists a prevailing practice in the broadcast and entertainment industries to treat talents like Sonza as
independent contractors. The right of labor to security of tenure as guaranteed in the Constitution arises only if there is an
employer-employee relationship under labor laws. Individuals with special skills, expertise or talent enjoy the freedom to offer
their services as independent contractors. The right to life and livelihood guarantees this freedom to contract as independent
contractors. The right of labor to security of tenure cannot operate to deprive an individual, possessed with special skills,
expertise and talent, of his right to contract as an independent contractor.

*Not every performance of services for a fee creates an employer-employee relationship. To hold that every person who
renders services to another for a fee is an employee – to give meaning to the security of tenure clause – will lead to absurd
results.

OROZCO vs THE FIFTH DIVISION OF THE COURT OF APPEALS

FACTS:

PDI engaged the services of Orozco to write a weekly column for its Lifestyle section. She religiously submitted her articles except
for a 6-month stint when she went to NY City. Nevertheless, she continued to send her articles through mail. She also received
compensation for every column that was published. When Orozco’s column appeared in the newspaper for last time, her editor,
Logarta, told her that the PDI’s editor-in-chief, Magsanoc, wanted to stop publishing her columns for no reason at all and advised
her to talk to the editor-in-chief. When Orozco talked to Magsanoc, the latter told her that it was the PDI chairperson who wanted
to stop the publication of her column. However, when Orozco talked to Apostol, the latter told her that Magsanoc informed her
that the Lifestyle section had already many columnists. PDI claims that Magsanoc met with the editor of the Lifestyle section to
discuss how to improve said section. They agreed to cut down the number of columnists by keeping only those whose columns
were well-written, with regular feedback and following. In their judgment, petitioner’s column failed to improve, continued to be
superficially and poorly written, and failed to meet the high standards of the newspaper. Hence, they decided to terminate
petitioner’s column. Orozco filed a complaint for illegal dismissal. The LA decided in favor of petitioner. On appeal, the NLRC
dismissed the appeal and affirmed the LA’s decision. The CA on the other hand, set aside the NLRC’s decision and dismissed
Orozco’s complaint.

ISSUE:

1. Whether petitioner is an employee of PDI.


2. Whether petitioner was illegally dismissed.

HELD:

Petition was dismissed. Judgment and Resolution affirmed.

Applying the four-fold test, the Court held that PDI lacked control over the petitioner. Though PDI issued guidelines for the
petitioner to follow in the course of writing her columns, careful examination reveals that the factors enumerated by the
petitioner are inherent conditions in running a newspaper. In other words, the so-called control as to time, space, and discipline
are dictated by the very nature of the newspaper business itself. Aside from the constraints presented by the space allocation of
her column, there were no restraints on her creativity; petitioner was free to write her column in the manner and style she was
accustomed to and to use whatever research method she deemed suitable for her purpose.

The apparent limitation that she had to write only on subjects that befitted the Lifestyle section did not translate to control but
was simply a logical consequence of the fact that her column appeared in that section and therefore had to cater to the preference
of the readers of that section. Orozco in this case is considered as an independent contractor. As stated in the case of Sonza vs.
ABS-CBN, independent contractors often present themselves to possess unique skills, expertise or talent to distinguish them from
ordinary employees. Like the petitioner in the cited case, Petitioner was engaged as a columnist for her talent, skill, experience,
and her unique viewpoint as a feminist advocate. How she utilized all these in writing her column was not subject to dictation by
respondent. As in Sonza, respondent PDI was not involved in the actual performance that produced the finished product. It only
reserved the right to shorten petitioner’s articles based on the newspaper’s capacity to accommodate the same. This fact was
not unique to petitioner’s column. It is a reality in the newspaper business that space constraints often dictate the length of
articles and columns, even those that regularly appear therein.

This Court has constantly adhered to the FOUR-FOLD TEST to determine whether there exists an employee-employer
relationship between parties. The four elements of an employee relationship are the selection and engagement of the
employee; the payment of wages; the power of dismissal; and the employer's power to control the employee's conduct.

Of these four elements, it is the power of control which is most crucial and most determinative factor, so important in fact the
the other elements may even be disregarded. In other words, the test is whether the employer controls or has reserved the
right to control the employee, not only as to the work done, but also as to the means and methods by which the same is
accomplished.

Furthermore, respondent PDI did not supply petitioner with the tools and instrumentalities she needed to perform her work.
Petitioner only needed her talent and skill to come up with a column every week. As such, she had all the tools she needed to
perform her work. Hence, since Orozco is not an employee of PDI, the latter cannot be held guilty of illegally dismissing the
petitioner.

MARIWASA vs LEOGARDO

FACTS:

Joaquin A. Dequila (or Dequilla) was hired on probation by Mariwasa Manufacturing, Inc. as a general utility worker on January
10, 1979. After 6 months, he was informed that his work was unsatisfactory and had failed to meet the required standards. To
give him another chance, and with Dequila’s written consent, Mariwasa extended Dequila’s probationary period for another
three months: from July 10 to October 9, 1979. Dequila’s performance, however, did not improve and Mariwasa terminated his
employment at the end of the extended period.
Dequila filed a complaint for illegal dismissal against Mariwasa and its VP for Administration, Angel T. Dazo, and violation of
Presidential Decrees Nos. 928 and 1389.

DIRECTOR OF MINISTRY OF LABOR: Complaint is dismissed. Termination is justified. Thus, Dequila appeals to the Minister of
Labor.

MINISTER OF LABOR: Deputy Minister Vicente Leogardo, Jr. held that Dequila was already a regular employee at the time of his
dismissal, thus, he was illegally dismissed. (Initial order: Reinstatement with full backwages. Later amended to direct payment
of Dequila’s backwages from the date of his dismissal to December 20, 1982 only.)

ISSUE:

WON employer and employee may, by agreement, extend the probationary period of employment beyond the six months
prescribed in Art. 282 of the Labor Code?

HELD:

YES, agreements stipulating longer probationary periods may constitute lawful exceptions to the statutory prescription
limiting such periods to six months.

The SC in its decision in Buiser vs. Leogardo, Jr. (1984) said that “Generally, the probationary period of employment is limited to
six (6) months. The exception to this general rule is when the parties to an employment contract may agree otherwise, such as
when the same is established by company policy or when the same is required by the nature of work to be performed by the
employee. In the latter case, there is recognition of the exercise of managerial prerogatives in requiring a longer period of
probationary employment, such as in the present case where the probationary period was set for eighteen (18) months, i.e. from
May, 1980 to October, 1981 inclusive, especially where the employee must learn a particular kind of work such as selling, or when
the job requires certain qualifications, skills experience or training.”

In this case, the extension given to Dequila could not have been pre-arranged to avoid the legal consequences of a probationary
period satisfactorily completed. In fact, it was ex gratia, an act of liberality on the part of his employer affording him a second
chance to make good after having initially failed to prove his worth as an employee. Such an act cannot now unjustly be turned
against said employer’s account to compel it to keep on its payroll one who could not perform according to its work standards.

By voluntarily agreeing to an extension of the probationary period, Dequila in effect waived any benefit attaching to the
completion of said period if he still failed to make the grade during the period of extension. By reasonably extending the period
of probation, the questioned agreement actually improved the probationary employee’s prospects of demonstrating his
fitness for regular employment.

ABBOT LABORATORIES vs ALCARAZ

FACTS:

The respondent Alcaraz was the Regulatory Affairs and Information Manager of Aventis Pasteur Philippines who showed interest
in applying as a Medical and Regulatory Affairs Manager, a position that was published by the petitioner Abbot Laboratories in
the newspaper. When the petitioner formally offered the position to the respondent, the latter accepted the position. It was on
May 23, 2005 that Walsh, Almazar and Bernardo formally handed to the respondent a letter terminating her employment with
the detailed explanation for her termination. The respondent then filed a complaint for illegal dismissal with damages against the
petitioner and its officers. The Labor Arbiter upheld the termination of probationary employment of the respondent holding that
the termination was justified with no evidence showing that the officers of the Abbot Lab acted in bad faith when terminating
her services.

The NLRC annulled and set aside the ruling of the Labor Arbiter which prompted the petitioners to file before the Court of Appeals
a petition for certiorari with prayer for issuance of a temporary restraining order and writ of preliminary injunction. Meanwhile,
the action of the petitioner on its motion for reconsideration of the CA’s resolution in the second CA petition was denied that
became final on January 10, 2011 because the petitioner failed to file a timely appeal on the said decision. Alcaraz, in her
comment, raised the issue on forum shopping when the petitioner filed its second petition to the CA pending the resolution of
the motion for reconsideration that they filed earlier in the December 10, 2009 decision. Alcaraz further contends that the
petitioners failed to comply with certification requirement under Section 5, Rule 7 of the rules of court when they failed to disclose
in their petition filed on June 16, 2010 Memorandum of Appeal filed before the NLRC.

ISSUE:

WON Alcaraz was sufficiently informed of the reasonable standards to qualify her as a regular employee

HELD:

YES. Abbott clearly conveyed to Alcaraz her duties and responsibilities as Regulatory Affairs Manager prior to, during the time of
her engagement, and the incipient stages of her employment. The employer is made to comply with two requirements when
dealing with a probationary employee:

1. The employer must communicate the regularization standards to the probationary employee
2. The employer must make such communication at the time of the probationary employee’s engagement

If the employer fails to comply with either, the employee is deemed as a regular and not a probationary employee. On this score,
the Court finds it apt to detail not only the incidents which point out to the efforts made by Abbott but also those circumstances
which would show that Alcaraz was well-apprised of her employer’s expectations that would, in turn, determine her
regularization. Alcaraz was well-aware that her regularization would depend on her ability and capacity to fulfill the requirements
of her position as Regulatory Affairs Manager and that her failure to perform such would give Abbott a valid cause to terminate
her probationary employment. Verily, basic knowledge and common sense dictate that the adequate performance of one’s
duties is, by and of itself, an inherent and implied standard for a probationary employee to be regularized; such is a
regularization standard which need not be literally spelled out or mapped into technical indicators in every case.

Keeping with [the Omnibus Rules Implementing the Labor Code], an employer is deemed to have made known the standards
that would qualify a probationary employee to be a regular employee when it has exerted reasonable efforts to apprise the
employee of what he is expected to do to accomplish during the trial of probation. This goes without saying that the employee
is sufficiently made aware of his probationary status as well as the length of time of the probation.

The exception to the foregoing is when the job is self-descriptive in nature, for instance, in the case of maids, cooks, drivers,
or messengers. Also in Aberdeen Court, Inc v. Agustin, it has been held that the rule on notifying a probationary employee of
the standards of regularization should not be used to exculpate an employee in a manner contrary to basic knowledge and
common sense in regard to which there is no need to spell out a policy or standard to be met. In the same light, an employee’s
failure to perform the duties and responsibilities which have been clearly made known to him constitutes a justifiable basis
for a probationary employee’s non-regularization.

ABS-CBN BROADCASTING CORPORATION vs. MARLYN NAZARENO

FACTS:

Petitioner ABS-CBN Broadcasting Corporation (ABS-CBN) is engaged in the broadcasting business and owns a network of
television and radio stations, whose operations revolve around the broadcast, transmission, and relay of telecommunication
signals. It sells and deals in or otherwise utilizes the airtime it generates from its radio and television operations. It has a franchise
as a broadcasting company, and was likewise issued a license and authority to operate by the National Telecommunications
Commission.

Petitioner employed respondents Nazareno, Gerzon, Deiparine, and Lerasan as production assistants (PAs) on different dates.
They were assigned at the news and public affairs, for various radio programs in the Cebu Broadcasting Station. On December
19, 1996, petitioner and the ABS-CBN Rank-and-File Employees executed a Collective Bargaining Agreement (CBA) to be effective
during the period from December 11, 1996 to December 11, 1999. However, since petitioner refused to recognize PAs as part of
the bargaining unit, respondents were not included to the CBA.
On October 12, 2000, respondents filed a Complaint for Recognition of Regular Employment Status, Underpayment of Overtime
Pay, Holiday Pay, Premium Pay, Service Incentive Pay, Sick Leave Pay, and 13th Month Pay with Damages against the petitioner
before the NLRC. The Labor Arbiter rendered judgment in favor of the respondents, and declared that they were regular
employees of petitioner as such, they were awarded monetary benefits. NLRC affirmed the decision of the Labor Arbiter.
Petitioner filed a motion for reconsideration but CA dismissed it.

ISSUE:

Whether or not the respondents were considered regular employees of ABS-CBN.

HELD:

The respondents are regular employees of ABS-CBN. It was held that where a person has rendered at least one year of service,
regardless of the nature of the activity performed, or where the work is continuous or intermittent, the employment is
considered regular as long as the activity exists, the reason being that a customary appointment is not indispensable before
one may be formally declared as having attained regular status.

In Universal Robina Corporation v. Catapang, the Court states that the primary standard, therefore, of determining regular
employment is the reasonable connection between the particular activity performed by the employee in relation to the usual
trade or business of the employer. The test is whether the former is usually necessary or desirable in the usual business or
trade of the employer. The connection can be determined by considering the nature of work performed and its relation to the
scheme of the particular business or trade in its entirety. Also, if the employee has been performing the job for at least a year,
even if the performance is not continuous and merely intermittent, the law deems repeated and continuing need for its
performance as sufficient evidence of the necessity if not indispensability of that activity to the business. Hence, the
employment is considered regular, but only with respect to such activity and while such activity exists.

Additionally, respondents cannot be considered as project or program employees because no evidence was presented to show
that the duration and scope of the project were determined or specified at the time of their engagement. In the case at bar,
however, the employer-employee relationship between petitioner and respondents has been proven. In the selection and
engagement of respondents, no peculiar or unique skill, talent or celebrity status was required from them because they were
merely hired through petitioner’s personnel department just like any ordinary employee. Respondents did not have the power
to bargain for huge talent fees, a circumstance negating independent contractual relationship. Respondents are highly dependent
on the petitioner for continued work. The degree of control and supervision exercised by petitioner over respondents through its
supervisors negates the allegation that respondents are independent contractors.

The presumption is that when the work done is an integral part of the regular business of the employer and when the worker,
relative to the employer, does not furnish an independent business or professional service, such work is a regular employment
of such employee and not an independent contractor. As regular employees, respondents are entitled to the benefits granted to
all other regular employees of petitioner under the CBA . Besides, only talent-artists were excluded from the CBA and not
production assistants who are regular employees of the respondents. Moreover, under Article 1702 of the New Civil Code: “In
case of doubt, all labor legislation and all labor contracts shall be construed in favor of the safety and decent living of the laborer.”

MARAGUINOT vs NLRC

FACTS:

Maraguinot and Enero were separately hired by Vic Del Rosario under Viva Films as part of the filming crew. Sometime in May
1992, sought the assistance of their supervisor to facilitate their request that their salary be adjusted in accordance with the
minimum wage law. On June 1992, Mrs. Cesario, their supervisor, told them that Mr. Vic Del Rosario would agree to their
request only if they sign a blank employment contract. Petitioners refused to sign such document. After which, the Mr. Enero
was forced to go on leave on the same month and refused to take him back when he reported for work. Mr. Maraguinot on the
other hand was dropped from the payroll but was returned days after. He was again asked to sign a blank employment contract
but when he refused, he was terminated.

Consequently, the petitioners sued for illegal dismissal before the Labor Arbiter. The private respondents claim the following:
(a) that VIVA FILMS is the trade name of VIVA PRODUCTIONS, INC. and that it was primarily engaged in the distribution &
exhibition of movies- but not then making of movies; (b) That they hire contractors called “producers” who act as independent
contractors as that of Vic Del Rosario; and (c) As such, there is no employee-employer relation between petitioners and private
respondents. The Labor Arbiter held that the complainants are employees of the private respondents. That the producers are
not independent contractor but should be considered as labor-only contractors and as such act as mere agent of the real
employer. Thus, the said employees are illegally dismissed.

The private respondents appealed to the NLRC which reversed the decision of the Labor Arbiter declaring that the complainants
were project employees due to the ff. reasons: (a) Complainants were hired for specific movie projects and their employment
was co-terminus with each movie project; (b)The work is dependent on the availability of projects. As a result, the total working
hours logged extremely varied; (c) The extremely irregular working days and hours of complainants work explains the lump sum
payment for their service; and (d) The respondents alleged that the complainants are not prohibited from working with other
movie companies whenever they are not working for the independent movie producers engaged by the respondents.

A motion for reconsideration was filed by the complainants but was denied by NLRC. In effect, they filed an instant petition
claiming that NLRC committed a grave abuse of discretion in: (a) Finding that petitioners were project employees; (b) Ruling
that petitioners were not illegally dismissed; and (c) Reversing the decision of the Labor Arbiter. In the instant case, the
petitioners allege that the NLRC acted in total disregard of evidence material or decisive of the controversy.

ISSUE:

WON there exist an employee- employer relationship between the petitioners and the private respondents.

HELD:

There exist an employee- employer relationship between the petitioners and the private respondents because of the ff. reasons
that nowhere in the appointment slip does it appear that it was the producer who hired the crew members. Moreover, it was
VIVA’s corporate name appearing on heading of the slip. It can likewise be said that it was VIVA who paid for the petitioners’
salaries.

Respondents also admit that the petitioners were part of a work pool wherein they attained the status of regular employees
because of the ff. requisites: (a) There is a continuous rehiring of project employees even after cessation of a project; (b) The
tasks performed by the alleged “project employees” are vital, necessary and indispensable to the usual business or trade of the
employer; and (c) However, the length of time which the employees are continually re-hired is not controlling but merely serves
as a badge of regular employment.

Citing Lao vs. NLRC, the could held that “a work pool may exist although the workers in the pool do not receive salaries and
are free to seek other employment during temporary breaks in the business, provided that the workers shall be available when
called to report for a project. Although primarily applicable to regular seasonal workers, this setup can likewise be applied to
project workers in so far as the effect of temporary cessation of work is concerned.”

Once a project or work pool employee has been (a) continuously, as opposed to intermittently, rehired by the same employer
for the same tasks or nature of tasks; (b) these tasks are vital, necessary, and indispensable to the usual business or trade of
the employer, then the employee must be deemed a regular employee, pursuant to Article 280 of the Labor Code and
jurisprudence.

Since the producer and the crew members are employees of VIVA and that these employees’ works deal with the making of
movies. It can be said that VIVA is engaged of making movies and not on the mere distribution of such.

LEYTE GEOTHERMAL POWER PROGRESSIVE EMPLOYEES UNION vs. PHILIPPINE NATIONAL OIL COMPANY – ENERGY
DEVELOPMENT CORPORATION

FACTS:

Respondent is a GOCC while petitioner is a legitimate labor organization. Among [respondent’s] geothermal projects is the Leyte
Geothermal Power Project located at the Greater Tongonan Geothermal Reservation in Leyte. Thus, the [respondent] hired and
employed hundreds of employees on a contractual basis, whereby, their employment was only good up to the completion or
termination of the project and would automatically expire upon the completion of such project.
Majority of the employees hired by [respondent] in its Leyte Geothermal Power Projects had become members of petitioner. In
view of that circumstance, the petitioner demands from the [respondent] for recognition of it as the collective bargaining agent
of said employees and for a CBA negotiation with it. However, the [respondent] did not heed such demands of the petitioner.
Sometime in 1998 when the project was about to be completed, the [respondent] proceeded to serve Notices of Termination of
Employment upon the employees who are members of the petitioner.

On December 28, 1998, the petitioner filed a Notice of Strike with DOLE against the [respondent] on the ground of purported
commission by the latter of unfair labor practice for “refusal to bargain collectively, union busting and mass termination.” On
the same day, the petitioner declared a strike and staged such strike. Secretary of Labor intervened and ordered all workers to
return to work. However, petitioner did not abide.NLRC: ruled that the employees are PROJECT EMPLOYEES, and the strike as
ILLEGAL Petitioner Union contends that its officers and members performed activities that were usually necessary and desirable
to respondent’s usual business.

ISSUE: WON they are project employees

HELD:

They are PROJECT EMPLOYEES

Article 280 of the Labor Code contemplates four (4) kinds of employees:

(a) regular employees or those who have been “engaged to perform activities which are usually necessary or desirable in the
usual business or trade of the employer”;
(b) project employees or those “whose employment has been fixed for a specific project or undertaking[,] the completion or
termination of which has been determined at the time of the engagement of the employee”;
(c) seasonal employees or those who work or perform services which are seasonal in nature, and the employment is for the
duration of the season; and
(d) casual employees or those who are not regular, project, or seasonal employees. Jurisprudence has added a fifth kind— a
fixed-term employee.

By entering into such a contract, an employee is deemed to understand that his employment is coterminous with the project.
He may not expect to be employed continuously beyond the completion of the project. It is of judicial notice that project
employees engaged for manual services or those for special skills like those of carpenters or masons, are, as a rule, unschooled.
However, this fact alone is not a valid reason for bestowing special treatment on them or for invalidating a contract of
employment. Project employment contracts are not lopsided agreements in favor of only one party thereto. The employer’s
interest is equally important as that of the employee[s’] for theirs is the interest that propels economic activity. While it may be
true that it is the employer who drafts project employment contracts with its business interest as overriding consideration, such
contracts do not, of necessity, prejudice the employee. Neither is the employee left helpless by a prejudicial employment
contract. After all, under the law, the interest of the worker is paramount.

Union’s own admission, both parties had executed the contracts freely and voluntarily without force, duress or acts tending to
vitiate the worker[s’] consent. Thus, we see no reason not to honor and give effect to the terms and conditions stipulated therein.

The litmus test to determine whether an individual is a project employee lies in setting a fixed period of employment involving
a specific undertaking which completion or termination has been determined at the time of the particular employee’s
engagement.

TOMAS LAO CONSTRUCTION vs NLRC

FACTS:

Private respondents were filed complaints for illegal dismissal against petitioners with NLRC. Respondents were hired for various
periods as construction workers in different capacities they described in the terms. They alternately worked for Tomas Lao Corp.,
Tomas and James Developer, LVM Construction, altogether as Lao Group of Companies. They engaged in construction of public
roads and bridges. Each one would also allow the utilization of the employees. With the arrangement workers were transferred
whenever necessary to on-going projects of the same company or rehired after the completion of the project or project phase
which they were assigned. In 1989 issued memorandum requiring all workers and company personnel to sign employment
contracts forms and clearances. To ensure compliance with the directive, the company ordered the withholding of the salary of
any employee who refused to sign. All respondents refused to sign contending that this scheme was designed by their employer
to downgrade their status from their regular employees to mere project employees. Their salaries were withheld. Since the
workers stood firm in their refusal to comply with the directives their services were terminated. The NLRC dismissed the complaint
finding that respondents were project employees whose employees could be terminated upon the completion of the project.
However the decision of LA was reversed on appeal finding that respondents were regular employees who were dismissed
without just cause and denied due process. The petitioners expostulation is that respondents have no valid cause to complain
about their employment contracts since documents formalized their status as project employees. They cite Policy Instruction No.
20 of DOLE which defines project employees as those employed in connection with particular construction project.

ISSUE:

WON the employees are regular or project employees

HELD:

The court ruled that, the principal test in determining whether particular employees are project employees distinguished from
regular employees is whether the project employees are assigned to carry out specific project or undertaking, the duration of
which are specified at the time of the employees are engaged for the project. Project in the realm of industry and business
refers to a particular job or undertaking that it is within the regular or usual business of employer, but which is distinct and
separate and identifiable as such from the undertakings of the company. They allowed to workers hired for specific projects
and hence can be classified as project employees, the repeated re-hiring and the continuing need for the services over a long
span of time have undeniably made them regular employees. Length of time may not be a controlling test for project
employment, it can be a strong factor in determining whether the employee was hired for a specific undertaking or in fact
tasked to perform functions which are vital, necessary and indispensable to the usual business or trade of the employer.

In the case at bar, private respondents had already gone through the status of project employees. But their employments became
non-coterminous with specific projects when they started to be continuously re-hired due to demands of petitioners business
and were re-engaged for many more projects without interruption. The denial by petitioners of the existence of a work pool in
the company because their projects were not continuous. A work pool may exist although the workers in the pool do not receive
salaries and are free to seek other employment during temporary breaks in the business, provided that the worker shall be
available when called to report for a project. The court finds that the continuous re- hiring of the same set of employees within
the framework is strongly indicative that private respondents were an integral part of a work pool in which petitioners drew its
workers for its various projects.

INDUSTRIAL-COMMERCIAL-AGRICULTURAL WORKERS ORGANIZATION (ICAWO), vs. COURT OF INDUSTRIAL RELATIONS (CIR)

FACTS:

Petitioners contend that they are regular and old employees and, as such, they should have been re-hired at the start, in the
month of October, of each milling season, which usually lasts 5 months. Respondents, on the other hand, urge that these laborers
are new, their employment terminating at the end of each milling season and, therefore, could not be re-admitted without the
company violating the closed-shop agreement with the CAPAWA. Held: Petitioners, even if seasonal workers, were not "new
workers" within the scope of the closed shop contract between the sugar central and the CAPAWA Union; hence, their discharge
was illegal.

ISSUE:

WON petitioners are regular or seasonal employees

HELD:

The arguments gravitate around the status of the seasonal workers, the petitioner contending that they are regular and old
employees and, as such, they should have been re-hired at the start, in the month of October, of each milling season, which
usually last 5 months. The respondents, on the other hand, urge that these laborers are new, their employment terminating at
the end of each milling season and, therefore, could not be re-admitted without the company violating the closed shop
agreement with the CAPAWA.

In an almost identical case, involving practically the same parties, G. R. No. L-17422, 28 February 1962, this Court interpreted
the closed shop agreement, jam quot, as referring "to future or new employees or laborers." This interpretation, however, does
not resolve the present issue because it does not classify the seasonal workers one way or the other. A direct precedent,
however, exists in the case of Manila Hotel Company v. Court of Industrial Relations, Et Al., L-18873, 30 September 1963,
wherein this Court, alluding to certain employees in the Pines Hotel in Baguio, stated:jgc:chanroble

". . . Their status is that of regular seasonal employees who are called to work from time to time, mostly during summer
season. The nature of their relationship with the hotel is such that during off season they are temporarily laid off but during
summer season they are re-employed, or when their services may be needed. They are not strictly speaking separated from
the service but are merely considered as on leave of absence without pay until they are re-employed. Their employment
relationship is never severed but only suspended. As such, these employees can be considered as in the regular employment
of the hotel."cralaw virtua 1aw library

GAPAYAO vs FULO

FACTS:

Jaime Fulo had been working in a farm owned by Jaime Gapayao since 1983. In November 1997, Jaime Fulo was electrocuted
while working in the said farm. Jaime Fulo died. Thereafter, Rosario Fulo, the widow of Jaime Fulo, filed a claim for death benefits
before the SSS (Social Security System). It turned out however that Jaime Fulo was never registered with the SSS. Eventually, SSS
ordered Gapayao, as the employer, to pay the SSS contributions due with penalty. Gapayao averred he cannot be made liable to
pay the SSS contributions because according to him there was no employer-employee relationship between him and Jaime Fulo.

He argued, among others, that Jaime Fulo was not his employee because:
1. he did not work regular hours as he was only called when needed and that Fulo can even look for other jobs elsewhere if he
wanted to. In fact, Fulo also worked for some other people;
2. he was only an “extra” in the farm;
3. Gapayao had no control over him (lack of control);

ISSUE:

Whether or not Jaime Fulo was an employee of Jaime Gapayao.

HELD:

Yes. Fulo was a regular employee and was thus entitled to receive SSS benefits, among others. The Supreme Court agreed with
the Court of Appeals in ruling that it “does not follow that a person who does not observe normal hours of work cannot be
deemed an employee.” It is also not material that Gapayao never supervised Fulo.

In this case, the number of hours worked is not material. Gapayao is considered a pakyaw worker. Pakyaw workers are
considered regular employees for as long as their employers have control over them. The power of the employer to control
the work of the employee is considered the most significant determinant of the existence of an employer-employee
relationship. This is the so-called control test and is premised on whether the person for whom the services are performed
reserves the right to control both the end achieved and the manner and means used to achieve that end.” It should be
remembered that the control test merely calls for the existence of the right to control, and not necessarily the exercise
thereof. It is not essential that the employer actually supervises the performance of duties by the employee. It is enough that
the former has a right to wield the power.
BRENT SCHOOL vs ZAMORA

FACTS:

Alegre was an athletic director at Brent, at a yearly compensation of P20,000. His contract fixed a specific term of 5 years for its
existence, from July, 1971, to July, 1976. Subsequent subsidiary agreements in March 1973, August 1973, and Sept. 1974
reiterated the same terms and conditions, including the expiry date, as those contained in the original contract of July, 1971.- 3
months before the expiration of the stipulated period, in April 1976, Alegre was given a copy of the report filed by Brent with the
Dep. of Labor advising of the termination of his services, effective July 16, 1976. The stated ground for termination was “completion of
contract, expiration of the definite period of employment.” Alegre protested the announced termination of his employment. He argued that
although his contract did stipulate that the same would terminate on July 17, 1976, since his services were necessary and
desirable in the usual business of his employer, and his employment had lasted for five years, he had acquired the status of
regular employee and could not be removed except for valid cause. The employment contract of 1971 was executed when the
Labor Code of the Philippines had not yet been promulgated, which came into effect some 3 years after the perfection of the
contract.

ISSUE:

WON the provisions of the Labor Code as amended (regarding probationary/regular employees), have anathematized "fixed
period employment" or employment for a term.

HELD:

No. Before the Labor Code, there was no doubt about the validity of term employment. It was impliedly but clearly recognized
by the Termination Pay law, RA 1052. The employment contract between Brent School and Alegre was executed on July 18, 1971,
at a time when the Labor Code of the Philippines (P.D. 442) had not yet been promulgated. Indeed, the Code did not come into
effect until November 1, 1974, some three years after the perfection of the employment contract, and rights and obligations
there under had arisen and been mutually observed and enforced. At that time, i.e., before the advent of the Labor Code, there
was no doubt whatever about the validity of term employment. It was impliedly but nonetheless clearly recognized by the
Termination Pay Law, R.A. 1052, 11 as amended by R.A. 1787. 12 Basically, this statute provided that

In cases of employment, without a definite period, in a commercial, industrial, or agricultural establishment or enterprise, the
employer or the employee may terminate at any time the employment with just cause; or without just cause in the case of an
employee by serving written notice on the employer at least one month in advance, or in the case of an employer, by serving
such notice to the employee at least one month in

In Brent School, Inc. v. Zamora the Court, for the first time,
recognized and resolved the anomaly created by a narrow and literal interpretation of Article 280 of the Labor Code that
appears to restrict the employee’s right to freely stipulate with his employer on the duration of his engagement. In this case, the
Court upheld the validity of the fixed-term employment agreed upon by the employer, Brent School, Inc., and the employee,
Dorotio Alegre, declaring that the restrictive clause in Article 280 “should be construed to refer to the substantive evil that
the Code itself x x x singled out: agreements entered into precisely to circumvent security of tenure. It should have no application
to instances where [the] fixed period of employment was agreed upon knowingly and voluntarily by the parties x x x absent any
x x x circumstances vitiating [the employee’s] consent, or where [the facts satisfactorily show] that the employer and [the]
employee dealt with each other on more or less equal terms[.]” The indispensability or desirability of the activity performed by
the employee will not preclude the parties from entering into an otherwise valid fixed term employment agreement; a definite
period of employment does not essentially contradict the nature of the employee’s duties as necessary and desirable to the usual
business or trade of the employer.

The SC recognized the fixed term type of employment embodied in the contract specifying that the services of the employee
shall be engaged only for a definite period, the termination of which occurs upon the expiration of the said period irrespective
of the existence of just cause and regardless of the activity the employee is called upon to perform. Considering, however, the
possibility of abuse by employers in the utilization of fixed-term employment contracts, the SC, in Brent, laid downthe
following criteria to prevent the circumvention of the employee’s security of tenure:

1. The fixed period of employment was knowingly and voluntarily agreed upon by the parties without any force, duress, or
improper pressure being brought to bear upon the employee and absent any other circumstances vitiating his consent; or
2. It satisfactorily appears that the employer and the employee dealt with each other more or less equal terms with no moral
dominance exercised by the former or the latter.

DIGITAL TELECOMMUNICATIONS vs DIGITEL

FACTS:

Digitel Employees Union and Digitel commenced collective bargaining negotiations which resulted in a bargaining deadlock. On
despite the order of the Labor Secretary to execute a CBA, still, no CBA was forged between Digitel and the Union. Some Union
members abandoned their employment with Digitel. The Union later became dormant. 10 years thereafter, Digitel received the
President of the Union, a letter containing the list of officers, CBA proposals and ground rules. Digitel was reluctant to negotiate
with the Union and demanded that the latter show compliance with the provisions of the Union’s Constitution and Bylaws on u
nion membership and election of officers.The faction filed a case for Preventive Mediation before the NCMB based on Digitel’s
violation of the duty to bargain. During the pendency, Interactive Technology Solutions, Inc. (Itech) was incorporated. Then, Lab
or Secretary assumed jurisdiction over the labor dispute.During the pendency of the controversy, Digitel Service, Inc. (Digiserv) f
iled with the DOLE an Establishment Termination Report stating that it will cease its business operation. The closure affected at
least 100 employees, 42 of whom are members of the herein respondent U

ISSUE:

WON Digiserv is a legitimate job contractor

HELD:

Labor-only contracting is expressly prohibited by our labor laws. Article 106 of the Labor Code defines labor-only contracting as
"supplying workers to an employer [who] does not have substantial capital or investment in the form of tools, equipment,
machineries, work premises, among others, and the workers recruited and placed by such person are performing activities which
are directly related to the principal business of such employer."

Section 5, Rule VIII-A, Book III of the Omnibus Rules Implementing the Labor Code (Implementing Rules), as amended by
Department Order No. 18-02, expounds on the prohibition against labor-only contracting, thus:

Section 5. Prohibition against labor-only contracting. − Labor-only contracting is hereby declared prohibited. For this purpose,
labor-only contracting shall refer to an arrangement where the contractor or subcontractor merely recruits, supplies or places
workers to perform a job, work or service for a principal, and any of the following elements are present:

i) The contractor or subcontractor does not have substantial capital or investment which relates to the job, work or service to be
performed and the employees recruited, supplied or placed by such contractor or subcontractor are performing activities which
are directly related to the main business of the principal; or ii) The contractor does not exercise the right to control over the
performance of the work of the contractual employee. The foregoing provisions shall be without prejudice to the application of
Article 248 (c) of the Labor Code, as amended. The "right to control" shall refer to the right reserved to the person for whom,
the services of the contractual workers are performed, to determine not only the end to be achieved, but also the manner and
means to be used in reaching that end. The law and its implementing rules allow contracting arrangements for the performance
of specific jobs, works or services. Indeed, it is management prerogative to farm out any of its activities, regardless of whether
such activity is peripheral or core in nature. However, in order for such outsourcing to be valid, it must be made to an
independent contractor because the current labor rules expressly prohibit labor-only contracting.

After an exhaustive review of the records, there is no showing that first, Digiserv has substantial investment in the form of capital,
equipment or tools. Under the Implementing Rules, substantial capital or investment refers to "capital stocks and subscribed
capitalization in the case of corporations, tools, equipment, implements, machineries and work premises, actually and directly
used by the contractor or subcontractor in the performance or completion of the job, work or service contracted out." The NLRC,
as echoed by the Court of Appeals, did not find substantial Digiserv’s authorized capital stock of One Million Pesos (₱
1,000,000.00). It pointed out that only Two Hundred Fifty Thousand Pesos (₱ 250,000.00) of the authorized capital stock had
been subscribed and only Sixty-Two Thousand Five Hundred Pesos (₱ 62,500.00) had been paid up. There was no increase in
capitalization for the last ten (10) years.19

Moreover, in the Amended Articles of Incorporation, as well as in the General Information Sheets for the years 1994, 2001 and
2005, the primary purpose of Digiserv is to provide manpower services. In PCI Automation Center, Inc. v. National Labor Relations
Commission,20 the Court made the following distinction: "the legitimate job contractor provides services while the labor-only
contractor provides only manpower. The legitimate job contractor undertakes to perform a specific job for the principal
employer while the labor-only contractor merely provides the personnel to work for the principal employer." The services
provided by employees of Digiserv are directly related to the business of Digitel, as rationalized by the NLRC in this wise:

Furthermore, Digiserv does not exercise control over the affected employees. The NLRC highlighted the fact that Digiserv shared
the same Human Resources, Accounting, Audit and Legal Departments with Digitel which manifested that it was Digitel who
exercised control over the performance of the affected employees. The NLRC also relied on the letters of commendation, plaques
of appreciation and certification issued by Digitel to the Customer Service Representatives as evidence of control. Considering
that Digiserv has been found to be engaged in labor-only contracting, the dismissed employees are deemed employees of Digitel.

Section 7 of the Implementing Rules holds that labor-only contracting would give rise to: (1) the creation of an employer-
employee relationship between the principal and the employees of the contractor or sub-contractor; and (2) the solidary
liability of the principal and the contractor to the employees in the event of any violation of the Labor Code.

Accordingly, Digitel is considered the principal employer of respondent employees.

PHILIPPINE BANK OF COMMUNICATION vs NLRC

FACTS:

Philippine Bank of Communications (PBC) and the Corporate Executive Search Inc. (CESI) entered into a letter agreement dated
January 1976 under which CESI undertook to provide temporary services to PBC consisting of the temporary services of 11
messengers. Attached to the letter was a list of messengers to be assigned, which included Ricardo Orpiada. Ricardo Orpiada was
thus assigned to work with the petitioner bank. As such, he rendered services to the bank, within the premises of the bank and
alongside other people also rendering services to the bank. There was some question as to when Ricardo Orpiada commenced
rendering services to the bank. As noted above, the letter agreement was dated January 1976. However, the position paper
submitted by (CESI) to the National Labor Relations Commission stated that (CESI) hired Ricardo Orpiada on 25 June 1975 as a
Temporary Service employee, and assigned him to work with the petitioner bank "as evidenced by the appointment memo issued
to him on 25 June 1975. "

October 1976, PBC requested CESI to withdraw Orpiada’s assignment because his services were no longer needed. Orpiada
instituted a complaint with the Dept of Labor for illegal dismissal and failure to pay 13th month pay. After investigation, the Office
of the Regional Director issued an order dismissing Orpiada’s complaint for failure to show existence of an employer-employee
relationship between the bank and himself. Orpiada succeeded in having his complaint certified for compulsory arbitration, CESI
was made an additional respondent. Labor Arbiter Dogelio rendered a decision ordering PBC to reinstated Orpiada to the same
or equivalent position with full back wages and to pay his 13th month pay. On appeal, NLRC modified the lBaor Arbiter’s decision
limiting back wages to two years and affirmed in all other aspects.

ISSUE:

WON EE-ER relationship existed between the bank and the respondent

HELD:

Yes, there was an employer-employee relationship. The court affirmed the NLRC decision.

With respect to the selection and engagement of the employee, although Orpiada was not personally selected by PBC, his
selection was still subject to the acceptance of the bank. With respect to wages, PBC remitted to CESI amounts corresponding to
the daily service rate and CESI paid the wages. Orpiada did not even appear in the payroll of PBC, but was listed in the payroll of
CESI. With respect to power of dismissal, after withdrawal from his assignment, he was also terminated by CESI. It would appear
that he was hired by CESI specifically for assignment with PBC. With regards to control, since Orpiada performed his functions
within the bank’s premises, not within CESI’s premises, he must have been subject to at least the same control and supervision
that the bank exercises. Application of the above factors in the specific context of this case appears to yield mixed results so far
as concerns the existence of an employer- employer relationship between the bank and Orpiada. Under the general rule set out
in the first and second paragraphs of Article 106, an employer who enters into a contract with a contractor for the performance
of work for the employer, does not thereby create an employer-employee relationship between himself and the employees
of the contractor. Thus, the employees of the contractor remain the contractor's employees and his alone. Nonetheless when
a contractor fails to pay the wages of his employees in accordance with the Labor Code, the employer who contracted out the
job to the contractor becomes jointly and severally liable with his contractor to the employees of the latter "to the extent of
the work performed under the contract" as such employer were the employer of the contractor's employees.

We hold that, in the circumstances 'instances of this case, (CESI) was engaged in "labor-only" or attracting vis-a-vis the petitioner
and in respect Ricardo Orpiada, and that because there is labor-only contracting, the petitioner bank is liable to Orpiada as if
Orpiada had been directly, employed not only by (CESI) but also by the bank. It may well be that the bank may in turn proceed
against (CESI) to obtain reimbursement of, or some contribution to, the amounts which the bank will have to pay to Orpiada; but
this it is not necessary to determine here.

NERI vs NLRC

FACTS:

Petitioners instituted complaints against FEBTC (Far East Bank and Trust Company) and BCC (Building Care Corporation) to compel
the bank to accept them as regular employees and for it to pay the differential between the wages being paid them by BCC and
those received by FEBTC employees with similar length of service. They contended that BCC in engaged in labor-only contracting
because it failed to adduce evidence purporting to show that it invested in the form of tools, equipment, machineries, work
premises and other materials which are necessary in the conduct of its business. Moreover, petitioners argue that they perform
duties which are directly related to the principal business or operation of FEBTC.

ISSUE:

Whether or not BCC was engaged in labor-only contracting.

HELD:

It is well-settled that there is labor-only contracting where: (a) the person supplying workers to an employer does not have
substantial capital or investment in the form of tools, equipment, machineries, work premises, among others; and, (b) the
workers recruited and placed by such person are performing activities which are directly related to the principal business of
the employer. BCC need not prove that it made investments in the form of tools, equipment, machineries, work premises, among
others, because it has established that it has sufficient capitalization. This fact was both determined by the Labor Arbiter and the
NLRC as BCC had a capital stock of P1 million fully subscribed and paid for. BCC is therefore a highly capitalized venture and cannot be deemed
engaged in labor-only contracting

VINOYA vs NLRC

FACTS:

Petitioner Vinoya was hired by RFC (Regent Food Corporation) as sales representative. He avers that he was transferred by RFC
to PMCI (Peninsula Manpower Corporation, Inc.), an agency which provides RFC with additional contractual workers. In PMCI,
he was reassigned to RFC as sales representative and then later informed by the personnel manager of RFC that his services
were terminated. RFC maintains that no employer-employee relationship existed between petitioner and itself. Petitioner filed
complaint for illegal dismissal. RFC alleges that PMCI is an independent contractor as the latter is a highly capitalized venture.

ISSUE:

WON petitioner was an employee of RFC and thereby, illegally dismissed.


HELD:

Yes. PMCI was a labor-only contractor. Although the Neri doctrine stated that it was enough that a contractor had substantial
capital to show it was an independent contractor, the case of Fuji Xerox clarified the doctrine stating that an independent
business must undertake the performance of the contract according to its own manner and method free from the control of
the principal. In this case, PMCI did not even have substantial capitalization as only a small amount of its authorized capital stock
was actually paid-in. Also, PMCI did not carry on an independent business or undertake the performance of its contract according
to its own manner and method. Furthermore, PMCI was not engaged to perform a specific and special job or service, which is
one of the strong indicators that is an independent contractor. Lastly, in labor-only contracting, the employees supplied by the
contractor perform activities, which are directly related to the main business of its principal. It is clear that in this case, the work
of petitioner as sales representative was directly related to the business of RFC. Since due to petitioner’s length of service, he
attained the status of regular employee thus cannot be terminated without just or valid cause. RFC failed to prove that his
dismissal was for cause and that he was afforded procedural due process. Petitioner is thus entitled to reinstatement plus full
backwages from his dismissal up to actual reinstatement.

ALIVIADO vs PROCTER AND GAMBLE

FACTS:

Petitioners worked as merchandisers of P&G. They all individually signed employment contracts with either Promm-Gem or
SAPS. They were assigned at different outlets, supermarkets and stores where they handled all the products of P&G. They
received their wages from Promm-Gem or SAPS.

SAPS and Promm-Gem imposed disciplinary measures on erring merchandisers for reasons such as habitual absenteeism,
dishonesty or changing day-off without prior notice.

To enhance consumer awareness and acceptance of the products, P&G entered into contracts with Promm-Gem and SAPS for
the promotion and merchandising of its products.

In December 1991, petitioners filed a complaint against P&G for regularization, service incentive leave pay and other benefits
with damages.

ISSUE:

WON P&G is the employer of petitioners.

HELD:

In order to resolve the issue of whether P&G is the employer of petitioners, it is necessary to first determine whether Promm-
Gem and SAPS are labor-only contractors or legitimate job contractors. Clearly, the law and its implementing rules allow
contracting arrangements for the performance of specific jobs, works or services. However, in order for such outsourcing to be
valid, it must be made to an independent contractor because the current labor rules expressly prohibit labor-only contracting.

To emphasize, there is labor-only contracting when the contractor or sub-contractor merely recruits, supplies or places
workers to perform a job, work or service for a principal and any of the following elements are present:

i) The contractor or subcontractor does not have substantial capital or investment which relates to the job, work or
service to be performed andthe employees recruited, supplied or placed by such contractor or subcontractor are
performing activities which are directly related to the main business of the principal; or

ii) The contractor does not exercise the right to control over the performance of the work of the contractual
Under the circumstances, Promm-Gem cannot be considered as a labor-only contractor. We find that it is a legitimate
independent contractor.

Considering that SAPS has no substantial capital or investment and the workers it recruited are performing activities which are
directly related to the principal business of P&G, we find that the former is engaged in “labor-only contracting”.

Where labor-only contracting exists, the Labor Code itself establishes an employer-employee relationship between the
employer and the employees of the labor-only contractor. The statute establishes this relationship for a comprehensive
purpose: to prevent a circumvention of labor laws. The contractor is considered merely an agent of the principal employer and
the latter is responsible to the employees of the labor-only contractor as if such employees had been directly employed by the
principal employer.

STAR PAER CORPORATION vs SIMBOL

FACTS:

Petitioner was the employer of the respondents. Under the policy of Star Paper the employees are:

1. New applicants will not be allowed to be hired if in case he/she has a relative, up to the 3rd degree of relationship, already
employed by the company.
2. In case of two of our employees (singles, one male and another female) developed a friendly relationship during the course
of their employment and then decided to get married, one of them should resign to preserve the policy stated above.

Respondents Comia and Simbol both got married to their fellow employees. Estrella on the other hand had a relationship with a
co-employee resulting to her pregnancy on the belief that such was separated. The respondents allege that they were forced to
resign as a result of the implementation of the said assailed company policy. The Labor Arbiter and the NLRC ruled in favor of
petitioner. The decision was appealed to the Court of Appeals which reversed the decision.

ISSUE:

Whether the prohibition to marry in the contract of employment is valid

HELD:

It is significant to note that in the case at bar, respondents were hired after they were found fit for the job, but were asked to
resign when they married a co-employee. Petitioners failed to show how the marriage of Simbol, then a Sheeting Machine
Operator, to Alma Dayrit, then an employee of the Repacking Section, could be detrimental to its business operations. Neither
did petitioners explain how this detriment will happen in the case of Wilfreda Comia, then a Production Helper in the Selecting
Department, who married Howard Comia, then a helper in the cutter-machine. The policy is premised on the mere fear that
employees married to each other will be less efficient. If we uphold the questioned rule without valid justification, the
employer can create policies based on an unproven presumption of a perceived danger at the expense of an employee’s right
to security of tenure.

Petitioners contend that their policy will apply only when one employee marries a co-employee, but they are free to marry
persons other than co-employees. The questioned policy may not facially violate Article 136 of the Labor Code but it creates a
disproportionate effect and under the disparate impact theory, the only way it could pass judicial scrutiny is a showing that it
is reasonable despite the discriminatory, albeit disproportionate, effect. The failure of petitioners to prove a legitimate
business concern in imposing the questioned policy cannot prejudice the employee’s right to be free from arbitrary
discrimination based upon stereotypes of married persons working together in one company.

In the Philippines we employ the standard of reasonableness of the company policy which is parallel to the bona fide occupational
qualification requirement. This was illustrated in the cases of Duncan Association of Detailman vs. Gaxo Wellcome (2004) and
PT&Tv. NLRC (1997). These cases instruct us that the requirement of reasonableness must be clearly established to uphold the
questioned employment policy. The employer has the burden to prove the existence of a reasonable business necessity. In the
case at bar, there is no a reasonable business necessity. The employees were hired after they found fit for the job, but were asked
to resign when they married a co-employee. Star Paper failed to show how the were marriages of the employees could be
detrimental to its business operations. The policy is premised on the mere fear that employees married to each other will be less
efficient.

Lastly, the absence of a statute expressly prohibiting marital discrimination in our jurisdiction cannot benefit the petitioners.
The protection given to labor in our jurisdiction is vast and extensive that we cannot prudently draw inferences from the
legislature’s silence that married persons are not protected under our Constitution and declare valid a policy based on a
prejudice or stereotype. Thus, for failure of petitioners to present undisputed proof of a reasonable business necessity, we rule
that the questioned policy is an invalid exercise of management prerogative. Corollary, the issue as to whether respondents
Simbol and Comia resigned voluntarily has become moot and academic.

In the case of Estrella, the petitioner failed to adduce proof to justify her dismissal. Hence, the Court ruled that it was illegal.

DOMINGO vs RAYALA

FACTS:

Ma. Lourdes T. Domingo (Domingo), then Stenographic Reporter III at the NLRC, filed a Complaint for sexual harassment against
Rayala, the chairman of NLRC. She alleged that Rayala called her in his office and touched her shoulder, part of her neck then
tickled her ears. Rayala argued that his acts does not constitute sexual harassment because for it to exist, there must be a
demand, request or requirement of sexual favor.

ISSUE:

Whether or not Rayala commit sexual harassment.

HELD:

Yes. The law penalizing sexual harassment in our jurisdiction is RA 7877. Section 3 thereof defines work-related sexual
harassment in this wise:

Sec. 3. Work, Education or Training-related Sexual Harassment Defined. – Work, education or training-related sexual harassment
is committed by an employer, manager, supervisor, agent of the employer, teacher, instructor, professor, coach, trainor, or any
other person who, having authority, influence or moral ascendancy over another in a work or training or education environment,
demands, requests or otherwise requires any sexual favor from the other, regardless of whether the demand, request or
requirement for submission is accepted by the object of said Act.

In a work-related or employment environment, sexual harassment is committed when:

(1) The sexual favor is made as a condition in the hiring or in the employment, re-employment or continued employment of
said individual, or in granting said individual favorable compensation, terms, conditions, promotions, or privileges; or the
refusal to grant the sexual favor results in limiting, segregating or classifying the employee which in a way would
discriminate, deprive or diminish employment opportunities or otherwise adversely affect said employee;
(2) The above acts would impair the employee’s rights or privileges under existing labor laws; or
(3) The above acts would result in an intimidating, hostile, or offensive environment for the employee.

Even if we were to test Rayala’s acts strictly by the standards set in Section 3, RA 7877, he would still be administratively liable.
It is true that this provision calls for a “demand, request or requirement of a sexual favor.” But it is not necessary that the
demand, request or requirement of a sexual favor be articulated in a categorical oral or written statement. It may be discerned,
with equal certitude, from the acts of the offender. Holding and squeezing Domingo’s shoulders, running his fingers across her
neck and tickling her ear, having inappropriate conversations with her, giving her money allegedly for school expenses with a
promise of future privileges, and making statements with unmistakable sexual overtones – all these acts of Rayala resound with
deafening clarity the unspoken request for a sexual favor.
DUNCAN ASSOCIATION vs GLAXO

FACTS:

Pedro A. Tecson was hired by respondent Glaxo Wellcome Philippines, Inc. as medical representative. As stipulated in the
contract signed and agreed by Tecson, The Glaxo provides that an employee is expected to inform management of any existing
or future relationship by consanguinity or affinity with co-employees or employees of competing drug companies. If
management perceives a conflict of interest or a potential conflict between such relationship and the employee’s employment
with the company, the management and the employee will explore the possibility of a “transfer to another department or
preparation for employment outside the company after six months.

Subsequently, Tecson entered into a romantic relationship with Bettsy, an employee of Astra Pharmaceuticals, a competitor of
Glaxo. In 1998,Tecson married Bettsy, whilst constantly reminded by the District Manager regarding the conflict of interest
which his relationship with Bettsy might engender. When Tecson failed to resolve the conflicting issue, Glaxo offered Tecson a
separation pay or to be transferred from Camarines to Butuan-Surigao-Agusan sales area to which the former refused to abide.
Aggrieved, Tecson filed a petition to the National Conciliation and Mediation Board (NCMB) which affirmed Glaxo’s policy as
valid. CA affirmed NCMB’s decision, hence, this petition.

ISSUE:

Whether Glaxo’s policy against its employees marrying employees from competitor companies is valid, and whether said policy
violates the equal protection clause of the Constitution.

HELD:

The Court finds no merit in the petition.

The prohibition against personal or marital relationships with employees of competitor companies upon Glaxo’s employees is
reasonable under the circumstances because relationships of that nature might compromise the interests of the company. In
laying down the assailed company policy, Glaxo only aims to protect its interests against the possibility that a competitor company
will gain access to its secrets and procedures. Glaxo possesses the right to protect its economic interests. The law also recognizes
that management has rights which are also entitled to respect and enforcement in the interest of fair play.

The company policy does not violate the equal protection clause. In the contractual provision and the policy in its employee
handbook, Glaxo does not impose an absolute prohibition against relationships between its employees and those of competitor
companies. Its employees are free to cultivate relationships with and marry persons of their own choosing. What the company
merely seeks to avoid is a conflict of interest between the employee and the company that may arise out of such relationships.

Glaxo has a right to guard its trade secrets, manufacturing formulas, marketing strategies, and other confidential programs and
information from competitors. The prohibition against pesonal or marital relationships with employees of competitor
companies upon Glaxo's employees is reasonable under the circumstances because relationships of that nature might
compromise the interests of the company. That Glaxo possesses the right to protect its economic interest cannot be denied.

It is the settled principle that the commands of the equal protection clause are addressed only to the state or those acting
under color of its authority. Corollarily, it has been held in a long array of US Supreme Court decisions that the equal
protection clause erects to shield against merely privately conduct, however, discriminatory or wrongful.

The company actually enforced the policy after repeated requests to the employee to comply with the policy. Indeed the
application of the policy was made in an impartial and even-handed manner, with due regard for the lot of the employee.
PT&T vs NLRC

FACTS:

PT&T (Philippine Telegraph & Telephone Company) initially hired Grace de Guzman specifically as “Supernumerary Project
Worker”, for a fixed period from November 21, 1990 until April 20, 1991 as reliever for C.F. Tenorio who went on maternity
leave. She was again invited for employment as replacement of Erlina F. Dizon who went on leave on 2 periods, from June 10,
1991 to July 1, 1991 and July 19, 1991 to August 8, 1991.

On September 2, 1991, de Guzman was again asked to join PT&T as a probationary employee where probationary period will
cover 150 days. She indicated in the portion of the job application form under civil status that she was single although she had
contracted marriage a few months earlier. When petitioner learned later about the marriage, its branch supervisor, Delia M.
Oficial, sent de Guzman a memorandum requiring her to explain the discrepancy. Included in the memorandum, was a
reminder about the company’s policy of not accepting married women for employment. She was dismissed from the company
effective January 29, 1992. Labor Arbiter handed down decision on November 23, 1993 declaring that petitioner illegally
dismissed De Guzman, who had already gained the status of a regular employee. Furthermore, it was apparent that she had
been discriminated on account of her having contracted marriage in violation of company policies.

ISSUE:

WON the alleged concealment of civil status can be grounds to terminate the services of an employee.

HELD:

Article 136 of the Labor Code, one of the protective laws for women, explicitly prohibits discrimination merely by reason of
marriage of a female employee. It is recognized that company is free to regulate manpower and employment from hiring to
firing, according to their discretion and best business judgment, except in those cases of unlawful discrimination or those
provided by law.

PT&T’s policy of not accepting or disqualifying from work any woman worker who contracts marriage is afoul of the right
against discrimination provided to all women workers by our labor laws and by our Constitution. The record discloses clearly
that de Guzman’s ties with PT&T were dissolved principally because of the company’s policy that married women are not
qualified for employment in the company, and not merely because of her supposed acts of dishonesty.

The government abhors any stipulation or policy in the nature adopted by PT&T. As stated in the labor code:

“ART. 136. Stipulation against marriage. — It shall be unlawful for an employer to require as a condition of employment or
continuation of employment that a woman shall not get married, or to stipulate expressly or tacitly that upon getting married, a
woman employee shall be deemed resigned or separated, or to actually dismiss, discharge, discriminate or otherwise prejudice
a woman employee merely by reason of marriage.”

The policy of PT&T is in derogation of the provisions stated in Art.136 of the Labor Code on the right of a woman to be free from
any kind of stipulation against marriage in connection with her employment and it likewise is contrary to good morals and public
policy, depriving a woman of her freedom to choose her status, a privilege that is inherent in an individual as an intangible and
inalienable right. The kind of policy followed by PT&T strikes at the very essence, ideals and purpose of marriage as an
inviolable social institution and ultimately, family as the foundation of the nation. Such policy must be prohibited in all its
indirect, disguised or dissembled forms as discriminatory conduct derogatory of the laws of the land not only for order but
also imperatively required.

APEX MINING CORPORATION vs NLRC

FACTS:

Anda mungkin juga menyukai